LSAT and Law School Admissions Forum

Get expert LSAT preparation and law school admissions advice from PowerScore Test Preparation.

 Administrator
PowerScore Staff
  • PowerScore Staff
  • Posts: 8916
  • Joined: Feb 02, 2011
|
#23775
Complete Question Explanation

Must Be True. The correct answer choice is (A)

The author concludes that the least efficiently run government bureaucracies are the ones that most commonly receive an increase in funds, because a legislator’s most common response to an inadequately provided public service is an increase of funding for it.

If the statements in the editorial are all true, it must also be true that there is a logical equivalence between inadequately provided public services and inefficiently run government bureaucracies. If the least efficiently run government bureaucracies were not most commonly the cause of inadequately provided public services, then the conclusion would not follow from the premise. Answer choice (A) is therefore correct.

Answer choice (A): This is the correct answer choice. See discussion above.

Answer choice (B): Just because the legislators boost the funding for the inadequately provided public service does not mean that they will necessarily boost the funding for the bureaucracy providing that service. It is perfectly plausible that the bureaucracy itself receives decreased levels of funding. This answer choice is incorrect.

Answer choice (C): It is not necessary that legislators repeatedly boost the funding for the public service in question: multiple funding increases would only be necessary if the initial funding increase does not solve the problem, and only if subsequent inefficiencies are discovered and repeatedly addressed. It is perfectly possible that one boost in funding is sufficient to resolve the inefficiency.

Answer choice (D): The effect of boosting the funding for a public service is outside the scope of this argument, which is only about the cause of the increase – i.e. the discovery of a service that is not being adequately provided. Answer choice (D) expresses a negative view of government procedure, which may seem tempting, but is not supported by the information contained in the stimulus and is therefore incorrect.

Answer choice (E): This answer choice is an exaggeration and is therefore incorrect. Never attempt to infer absolute statements from information about relative claims.
 bk1111
  • Posts: 103
  • Joined: Apr 22, 2017
|
#45809
I initially chose A, but changed my answer to D upon blind review. Can someone elaborate on why the answer is not D? When reviewing, I presumed the stimulus was explaining the effect of boosting funds for a public service because it says "because of this"... Maybe I am just misinterpreting the sentence structure of it all but I want to make sure I don't make the same mistake going forward.
 Adam Tyson
PowerScore Staff
  • PowerScore Staff
  • Posts: 5153
  • Joined: Apr 14, 2011
|
#46958
Answer D has the chronology backwards, bk1111. It's not that the ones that get the money become less efficient, but the ones that are already inefficient then get the money.

The stimulus links adequacy of a service to funding - an inadequate service is found, and then money is added.

The conclusion is that inefficient services get the money - an inefficient service is found, and then money is added.

The correct answer will link inadequacy to inefficiency - inadequate services must be inefficient, or inefficient services will be inadequate.

Answer D posits a causal relationship between the funding and the inefficiency, failing to link inadequacy to inefficiency as well as getting the timing backwards.

I hope that helps clear it up!

Get the most out of your LSAT Prep Plus subscription.

Analyze and track your performance with our Testing and Analytics Package.